Sunteți pe pagina 1din 8

Math 324, Fall 2011

Assignment 8
Solutions

Exercise 1.(a) Let a and b be integers not divisible by the prime p. Show that either one or all three of the
integers a, b, and ab are quadratic residues of p.
(b) Show that if n has the prime factorization
2t
n = p2t
1
1 +1
· · · pk2tk +1 pk+1
k+1
· · · p2t
l
l

then      
n p1 pk
= ··· .
q q q

(c) Show that if b is a positive integer not divisible by the prime p then

p−1  
X ib
= 0.
i=1
p

Solution :(a) By multiplicativity of the Legendre symbol, one has


    
ab a b
= .
p p p
 
ab
Recalling that the Legendre symbol take values in ±1, multiplication by p yields
   
ab a b
1= .
p p p

In particular, the number of occurences of −1 on the right must be even, hence is either 0 or 2. This states
that 1 occurs either 3 or 1 times. In the first case, all three numbers are quadratic residues of p, while in the
second case exactly one of them is a quadratic residue of p.
(b) Using the multiplicativity of the Legendre symbol,
2tk+1
!
p2t1 +1
· · · pk2tk +1 pk+1 · · · p2tl
 
n 1 l
=
q q
2tk 2tk+1
!
2t1
p1 · · · pk pk+1 · · · p2t l

l p1 · · · pk
= .
q q

The first factor is equal to 1, since


 2
2tk 2tk+1 2tl tk tk+1 tl
p2t
1
1
· · · p k p k+1 · · · p l = p t1
1 · · · p p
k k+1 · · · p l

is a square. Thus,        
n p1 · · · pk p1 pk
= = ··· ,
q q q q
the last equality following from the multiplicativity of the Legendre symbol.
(c) Since b is not divisible by p, p being prime ensures that gcd(b, p) = 1. It follows that

b, 2b, · · · , (p − 1)b (1)

1
is a reduced residue system modulo p. Indeed. each residue ib is reduced, being the product of the reduced
residues i and b. On the other hand, if ib ≡ jb mod p then the fact b is reduced modulo p allows us to
conclude i ≡ j mod p. If 1 ≤ i, j ≤ p − 1, the last congruence implies that i = j. It follows that the numbers
in (1) are incongruent modulo p. Since p has exactly p − 1 reduced residue classes, the claim follows.
From class, we know that up to congruence there are (p − 1)/2 quadratic residues of p and (p − 1)/2
quadratic non-residues of p. Since the elements ib, 1 ≤ i ≤ p − 1, run over all reduced residues of p, we
deduce that (ib/p) = 1 for (p − 1)/2 choices of i and = −1 for the remaining (p − 1)/2 values of i. Thus,

p−1
X 
ib p−1 p−1
= − = 0.
i=1
p 2 2

Exercise 2. Consider the quadratic congruence

aX 2 + bX + c ≡ 0 mod p, (1)

where p is prime and a, b, and c are integers with a not divisible by p.


(a) Let p = 2 Determine which quadratic congruences modulo 2 have solutions.
(b) Let p be an odd prime and set d = b2 − 4ac. Show that the congruence (1) is equivalent to the congruence

Y 2 ≡ d mod p

where Y = 2aX + b. Determine the number of incongruent solutions modulo p of (1). (Hint : The last
answer will depend on d.)
Solution : (a) If p = 2 then we may assume a = 1 and b, c are either 0 or 1. There are thus four cases to
consider.
X 2 ≡ 0 mod 2
X 2 + X ≡ 0 mod 2
X 2 + 1 ≡ 0 mod 2
X 2 + X + 1 ≡ 0 mod 2
The first two congruences admit the solution X = 0, while the third has the solution X = 1. By inspection,
neither 0 nor 1 is a solution of the fourth congruence. Since any element is congruent to either 0 or 1
modulo 2, we see it has no solution.
Observing that the product of the constant and linear coefficients are 0 in the first three cases, and 1 in
the fourth case, we deduce that the general quadratic congruence

aX 2 + bX + c ≡ 0 mod 2, gcd(a, 2) = 1,

has a solution precisely when bc ≡ 0 mod 2.


(b) If
ax2 + bx + c ≡ 0 mod p
then
y 2 = (2ax + b)2 = 4a2 x2 + 4abx + b2 = 4a(ax2 + bx) + b2 ≡ 4a(−c) + b2 = d mod p.
On the other hand, suppose
y 2 ≡ d mod p.
By hypothesis, a is reduced modulo p. Since p is odd, 2 is also reduced modulo p, hence 2a is reduced
modulo p. The elementary theory of linear congruences asserts there is a unique x modulo p such that

y ≡ 2ax + b mod p.

2
For such x,

0 ≡ y 2 − d = (2ax + b)2 − (b2 − 4ac) = 4a2 x2 + 4abx + b2 − b2 + 4ac = 4a ax2 + bx + c mod p.




Observing that 4a = 2 · 2a is reduced modulo p, it can be cancelled to yield

0 ≡ ax2 + bx + c mod p.

The preceding shows that x is a soluton of (1) if and only if y = 2ax+b is a solution of the congruence y 2 ≡
d mod p, that is the congruences are equivalent.
The congruence
Y 2 ≡ 0 mod p
has the unique solution 0. The preceding discussion allows us to conclude that (1) has a unique solution
if d = 0. If d 6= 0 then
Y 2 ≡ d mod p
has either no solution if (d/p) = −1 or exactly two solutions if (d/p) = 1. Once again the preceding discussion
allows us to conclude that (1) has either no solution or exactly two according to d being a quadratic non-
residue or quadratic residue of p.

Exercise 3. Let p be a prime with p ≥ 7.


(a) Show that there are always two consecutive quadratic residues of p. (Hint : First show that at least one
of 2, 5, and 10 is a qudratic residue of p.)
(b) Show that there are always two quadratic residues of p that differ by 3.
Solution :(a) Since p 6= 2, 5, exercise 1(a) asserts that at least one of 2, 5 or 10 is a quadratic residue of p.
On the other hand, since p 6= 3, 1, 4,and 9 are all quadratic residues of p. It follows immediately that one of
the pairs (1, 2), (4, 5), or (9, 10) are consecutive quadratic residues of p.
(b) One can take 1 and 4.

Exercise 4. Let n be an odd integer with prime factorization


αk
n = pα
1 · · · pk .
1

Find an expression for the number of solutions of the congruence

X 2 ≡ a mod n
   
in terms of the Legendre symbols pa1 , . . ., pak .
Solution : We assume gcd(a, n) = 1.
By the Chinese Remainder Theorem, the solutions of the congruence

X 2 ≡ a mod n (1)

are in one-one correspondence to the solutions of the system of congruences

Yi2 ≡ a mod pα
i ,
i
1 ≤ i ≤ k. (2)

Observing gcd(a, n) = 1 implies gcd(a, pi ) = 1, the congruence

Yi2 ≡ a mod pi

has a solution if and only if a is a quadratic residue of pi . Furthermore, it has exactly two distinct non-zero
solutions modulo p, say ±bi . Observing
d(Yi2 − a)
= 2Yi
dYi

3
does not vanish at ±bi , Hensel’s Lemma ensures that each of the solutions lift to a unique solution of the
congruence
Yi2 ≡ a mod pαi .
i

Thus, the last congruence either has no solutions or precisely two incongruent solutions modulo pα
i , according
i

to whether a is a quadratic nonresidue or residue of pi .


Observing   
a 0, if a is a quadratic nonresidue of pi ;
1+ =
pi 2, if a is a quadratic residue of pi ,
the preceding discussion allows us to conclude that the system (2) has precisely
     
a a
1+ ··· 1 + .
p1 pk

solutions. The remarks at the start allows us to conlcude that this is also the number of solutions of the
congruence (1).

Exercise 5. (a) Show that if p is an odd prime then


  
3 1, if p ≡ ±1 mod 12
=
p −1 if p ≡ ±5 mod 12.

(Hint : Law of Quadratic Reciprocity.)


(b) Find a congruence describing all (odd) primes for which 5 is a qudratic residue.
Solution : If p is an odd prime other than 3, the Law of Quadratic Reciprocity yields
 
3 p
= (−1)(p−1)/2 .
p 3

From the last equation, ( 3p ) is seen to be 1 precisely when both factors on the left are equal. Recalling
that up to congruence 1 is the only quadratic residue of 3, if both factors are equal to 1 then

p ≡ 1 mod 4

and
p ≡ 1 mod 3,
hence the Chinese Remainder Theorem yields

p ≡ 1 mod 12.

On the other hand, as up to congruence 2 is the unique non-quadratic residue of 3, if both factors are equal
to −1 then
p ≡ 3 ≡ −1 mod 4
and
p ≡ 2 ≡ −1 mod 3,
hence the Chinese Remainder Theorem yields

p ≡ −1 mod 12.

In summary, ( p3 ) = 1 precisely when p ≡ ±1 mod 12.

4
On the other hand, ( p3 ) is equal to −1 precisely when exactly one of the factors is positive and the other
is negative. If (−1)(p−1)/2 = 1 and ( p3 ) = −1 then

p ≡ 1 ≡ 5 mod 4

and
p ≡ 2 ≡ 5 mod 3
hence the Chinese Remainder Theorem yields

p ≡ 5 mod 12.

On the other hand, if (−1)(p−1)/2 = −1 and ( p3 ) = 1 then

p ≡ 3 ≡ −5 mod 4

and
p ≡ 1 ≡ −5 mod 3
hence the Chinese Remainder Theorem yields

p ≡ −5 mod 12.

In summary, ( p3 ) = −1 precisely when p ≡ ±5 mod 12.


(b) Since 5 ≡ 1 mod 4, if p is an odd prime other than 5 then the Law of Quadratic Reciprocity yields
   
5 p
= .
p 5

Observing that 1 and 4 ≡ −1 mod 5 are the quadratic residues of 5, we deduce 5 is a quadratic residue of p
if and only if p ≡ ±1 mod 5.
7 21 107 37 11
    
Exercise 6. Evaluate the following Legendre symbols. (a) 53 . (b) 101 . (c) 1009 . (d) 641 . (e) 79 .
Solution :(a) Both 7 and 53 are prime. Observing 53 ≡ 1 mod 4, the Law of Quadratic Reciprocity yields
     
7 53 4
= = = 1.
53 7 7
.
(b) By multiplicativity of the Legendre symbol,
    
21 3 7
= .
101 101 101

Since 101 ≡ 1 mod 4, the Law of Quadratic Reciprocity yields


     
3 101 2
= = = −1
101 3 3

and      
7 101 3
= = = −1
101 7 7
by Exercise 5(a), since 7 ≡ −5 mod 12. Therefore,
    
21 3 7
= = (−1) · (−1) = 1.
101 101 101

5
(c) Both 107 and 1009 are prime. Since 1009 ≡ 1 mod 4, the Law of Quadratic Reciprocity yields
        
107 1009 46 2 23
= = = .
1009 107 107 107 107

Observing 107 ≡ 3 mod 8, we have  


2
= −1.
107
Furthermore, since both 23 and 107 are congruent to 3 modulo 4, the Law of Quadratic Reciprocity yields
   
23 107
=− = − (15 · 23) = − (5 · 23) (3 · 23) .
107 23
5

Since 23 6≡ ±1 mod 5, exercise 5(b) allows us to conclude that 23 = −1. Since 23 ≡ −1 mod 12, exer-
3

cise 5(a) allows us to conclude 23 = 1, hence
 
23
= −(−1) · 1 = 1.
107

In light of the preceding calculations, we deduce


    
107 2 23
= = −1 · 1 = −1.
1009 107 107

(d) As both 37 and 641 are prime, with the later congruent to 1 modulo 4, the Law of Quadratic Reciprocity
yields           
37 641 12 4 3 3
= = = = = 1,
641 37 37 37 37 37
by exercise 5(a), since 37 ≡ 1 mod 12.
(e) Observing 11 and 79 are both prime and congruent to 3 modulo 4, the Law of Quadratic Reciprocity
yields      
11 79 2
=− =− = −(−1) = 1,
79 11 11
since 11 ≡ 3 mod 8.

Bonus Question. (a) Show that Euler’s form of the Law of Quadratic Reciprocity implies the Law of
Quadratic Reciprocity as stated in class.
(b) Prove
  Euler’s form of the Law of Quadratic Reciprocity using Gauss’s Lemma. (Hint : Show that to
a
find p , one need only find the parity of the number of integers k satisfying one of the inequalities

(2t − 1)p tp
≤k≤ , t = 1, 2, . . . , 2u − 1,
2a a
where u = a/2 if a is even and u = (a − 1)/2 if a is odd. Then, take p = 4am + r with 0 < r < 4a and show
that finiding the parity of the number of integers k satisfying one of the inequalities is the same as finding
the parity of the number of integers satisfying one of the inequalities

(2t − 1)r tr
≤k≤ , t = 1, 2, . . . , 2u − 1.
2a a
Show that this number depends only on r. Then repeat the last step of the argument with r replaced
by 4a − r.)
Solution :(a) Let p and q be odd primes. We distinguish two cases

6
(I) p ≡ q mod 4 : Writing
p − q = 4a,
the fact p and q are distinct primes ensures that p does not divide a. In light of the properties of the
Legendre symbol, Euler’s form yields
                  
p p−q 4a a a 4a p−q −q −1 q
= = = = = = = = .
q q q q p p p p p p

If p ≡ 1 mod 4 then  
−1
= (−1)(p−1)/2 = 1 = (−1)(p−1)/2·(q−1)/2 .
p
On the other hand, if p ≡ 3 mod 4 then, recalling that p ≡ q mod 4 then
 
−1
= (−1)(p−1)/2 = −1 = (−1)(p−1)/2·(q−1)/2 .
p

In all cases, we see that  


−1
= (−1)(p−1)/2·(q−1)/2 ,
p
hence the preceding calculations yield
   
p q
= (−1)(p−1)/2·(q−1)/2 .
q p

(II) p ≡ −q mod 4 : In this case, one of p and q is congruent to 1 modulo 4, so

(−1)(p−1)/2·(q−1)/2 = 1.

Writing p+q = 4a, the fact p and q are distinct odd primes ensure that a is not divisible by p. Therefore,
Euler’s form yields
                 
p p+q 4a a a 4a p+q q q
= = = = = = = = (−1)(p−1)/2·(q−1)/2 .
q q q q p p p p p
 
a
(b) To apply Gauss’s Lemma to compute p , we need to find the parity of the numbers

p−1
a, 2a, . . . , a
2
which have least positive residue between p/2 and p. If ka is such a number then there exists an integer t
such that
p−1 (2t − 1)p
tp ≥ ka ≥ tp − = . (1)
2 2
Since 1 ≤ k ≤ (p − 1)/2, t must lie among

1, 2, . . . , ⌊a/2⌋ = u.

For each such t, dividing the inequality (1) by a yields

tp (2t − 1)p
≥k≥ 1≤t≤u (2)
a 2a
Thus we must find the parity of the number of integers k satisfying the last inequalities.

7
Suppose p = 4am + r, with 0 < r < 4a. The conditions (2) become

tr (2t − 1)r
4mt + ≥ k ≥ 2(2t − 1)m + , 1 ≤ t ≤ u.
a 2a
Since we are only interested in parity of the number of k, we can drop the even integers in each of these
inequalities (which reduces the number of k be an even number). Thus, the conditions reduce to

tr (2t − 1)r
≥k≥ ,
a 2a
which only depends on r. Therefore, if p ≡ q ≡ r mod 4a then
   
a a
= .
p q

In the case p ≡ −q ≡ r mod 4a, then q ≡ 4a − r mod 4a. Substituting 4a − r for r in the conditions we
get
t(4a − r) (2t − 1)(4a − r)
≥k≥
a 2a
which reduces to
tr (2t − 1)r
4t − ≥ k ≥ (2t − 1)2 − .
a 2a
Again, we may drop the even integers in each inequality, and multiplying through by −1 doesn’t change the
number of k, but it makes the conditions identitcalto the conditions for p above. Therefore,
   
a a
= .
p q

S-ar putea să vă placă și